Может ли ZFC решать теорию чисел?

Среди версий теоремы о неполноте, которые я видел, следующие:

  • Предполагая, что аксиомы Пеано непротиворечивы (а они таковы, если мы допускаем существование натуральных чисел), на языке теории чисел есть утверждения, которые нельзя доказать или опровергнуть с помощью аксиом Пеано.
  • Предполагая, что ZFC непротиворечива, на языке теории множеств есть утверждения, которые нельзя доказать или опровергнуть с помощью ZFC.

Что мне не совсем ясно, так это статус следующего утверждения:

  • Предполагая, что ZFC непротиворечива, на языке теории чисел существуют утверждения, теоретико-множественная интерпретация которых не может быть доказана или опровергнута с помощью ZFC.

Поскольку не каждое утверждение в теории множеств является интерпретацией утверждения в теории чисел, это, кажется, не следует из первых двух.

Ответы (3)

Доказательство Гёделя дает явную конструкцию для утверждения в данной (достаточно сильной и рекурсивно аксиоматизируемой) теории, которое не может быть доказано или опровергнуто в этой теории.

Случается так , что когда теория является ZFC, независимое утверждение оказывается тем, которое является теоретико-множественным представлением чисто теоретико-числового утверждения. По крайней мере, это тот случай, когда вы самым естественным образом устанавливаете, что ZFC удовлетворяет условию «достаточно сильного».

По сути, «достаточно сильный» сводится к способности представлять определенные основные теоретико-числовые конструкции, и тогда гёделевское предложение строится как представление определенного теоретико-числового свойства.

Так что ответ нет. Гёделевское предложение PA определяется ZFC, но гёделевское предложение ZFC, которое «случайно» оказалось предложением PA, — нет. Как это согласуется с тем, что ZFC предоставляет модель PA? Можно было бы подумать, что модель должна решать все предложения. Это потому, что ZFC предоставляет несколько нестандартных моделей, которые расходятся в независимых предложениях? Существует ли какая-нибудь известная пара теорий X и Y, обе подчиняющиеся условиям Гёделя, такая, что Y решает все предложения в X? Это вообще возможно?

Формальное утверждение «Con(ZFC)» является теоретико-числовым утверждением, таким же образом, как и «Con(PA)», и (при условии непротиворечивости ZFC) оно не зависит от ZFC.

Можно построить доказательство с помощью небольшого варианта доказательства разрешимости рекурсивно аксиоматизированной полной теории.

1 . Хорошо известно, что не существует алгоритма, который по любому предложению ф теории чисел, определит, верно ли это предложение в натуральных числах. Точнее, если набор предложений теории чисел индексируется как ( ф н ) любым из обычных способов, то множество н такой, что ф н верно в Н не является рекурсивным. На самом деле дело обстоит гораздо хуже: множество даже не арифметическое.

2. Предположим, что Т является рекурсивно аксиоматизированным расширением арифметики Пеано. Тогда есть приговор ф теории чисел так, что ф ни доказуема, ни опровергнута в Т . Идея состоит в том, что мы можем написать программу, которая перечисляет доказательства в Т , поскольку множество (индексов) доказательств рекурсивно перечислимо.

Если для каждого предложения ф теории чисел, один из ф или ¬ ф является теоремой Т , то рано или поздно один из ф или ¬ ф будет отображаться как последнее предложение доказательства в списке. Эта процедура дала бы алгоритм определения истинности предложений в Н , противоречит ( 1 ).

Есть небольшая техническая особенность, с которой нужно разобраться, поскольку ZFC не является расширением арифметики Пеано. Чтобы справиться с этим, для любого предложения ψ теории чисел, мы можем механически составить предложение ψ теории множеств такое, что если ψ является теоремой ZFC, то ψ верно в Н . Предложение ψ получается из обычного построения Н , а также его сложение и умножение в ZFC.